数学杂志  2017, Vol. 37 Issue (3): 474-480   PDF    
扩展功能
加入收藏夹
复制引文信息
加入引用管理器
Email Alert
RSS
本文作者相关文章
玄海燕
史永侠
张玉春
徐广业
基于ARFIMA-GARCH模型的混成检验
玄海燕1, 史永侠2, 张玉春1, 徐广业1     
1. 兰州理工大学经济管理学院, 甘肃 兰州 730050;
2. 兰州理工大学理学院, 甘肃 兰州 730050
摘要:本文研究了ARFIMA-GARCH模型的混成检验问题.基于拟极大指数似然估计,给出了平方残差自相关函数的渐近性,进而建立了基于平方残差自相关函数的混成检验统计量.通过实例分析,表明可利用基于平方残差自相关函数的混成检验统计量来诊断检验由拟极大指数似然估计方法拟合的ARFIMA-GARCH模型.
关键词ARFIMA-GARCH模型    混成检验    拟极大指数似然估计    
PORTMANTEAU TEST BASED ON THE ARFIMA-GARCH MODEL
XUAN Hai-yan1, SHI Yong-xia2, ZHANG Yu-chun1, XU Guang-ye1     
1. School of Economics and Management, Lanzhou University of Technology, Lanzhou 730050, China;
2. School of Sciences, Lanzhou University of Technology, Lanzhou 730050, China
Abstract: In this paper, we study the portmanteau test problem of ARFIMA-GARCH model. Based on the quasi-maximum exponential likelihood estimator, the asymptotic of squared residual autocorrelation function is given, and the portmanteau test statistic based on squared residual autocorrelation function is established. By the analysis of a real example, it is showed that we can use the portmanteau test statistic based on squared residual autocorrelation function to the diagnostic test of ARFIMA-GARCH model fitting by quasi-maximum exponential likelihood estimator.
Key words: ARFIMA-GARCH model     portmanteau test     quasi-maximum exponential likelihood estimation    
1 引言

在金融领域中, 大量时间序列数据具有丰富的特征, 如股票收益具有长期记忆性, 收益分布具有尖峰、细尾等[1], %如波动的长期记忆以及波动率结构变化等, 而这些特征有助于投资者选择最佳投资组合, 以便减少风险.为了考虑金融资产收益率序列的长记忆性, Hosking[2]提出了ARFIMA模型, 但ARFIMA模型不能够刻画资产收益率中普遍存在的波动聚类, 波动率是资产收益不确定性的衡量, 常被用来衡量资产的风险.Bollerslev[3]提出的GARCH模型能有效地捕捉资产收益率波动集聚现象.为了同时描述资产收益率中的长记忆性和异方差性, 因此产生了ARFIMA-GARCH模型, 该模型有利于投资者研究金融资产组合的风险水平问题.

然而, 许多工作致力于模型的提出以及参数估计, 对于模型诊断检验没有得到相应的重视.对于实践者来说, 去检验模型的准确性是一个基本的问题.在时间序列模型中, 混成检验是一种广泛有用的诊断工具, 特别对于拟合的模型.近年来, 随着金融市场日益变得复杂, 关于混成检验也相继有了一些较为成熟的结果.对于拟极大指数似然估计, Li[4]和Zhu[5]分别给出不同的混成检验统计量.基于高斯拟极大似然估计, Wong和Ling[6]研究了混合混成检验统计量.

本文对于ARFIMA-GARCH模型, 在拟极大指数似然估计下, 给出了平方残差自相关函数的渐近性, 并进行了证明, 进而得到了基于平方残差自相关函数的混成检验统计量; 由实例分析可知, 该统计量有利于诊断检验由拟极大指数似然估计拟合的ARFIMA-GARCH模型, 以便确定在实际的股票市场中, 拟合的模型是否准确, 确保对未来股票市场的变化做出较准确的预测.

2 ARFIMA-GARCH模型

ARFIMA $(p, d, q)$-GARCH $(r, s)$模型形式如下

$ \begin{eqnarray} \Phi(B)(1-B)^{d}y_{t}=\Psi(B)\varepsilon_{t}, \\ \end{eqnarray} $ (2.1)
$ \begin{eqnarray} \varepsilon_{t}=\eta_{t}\sqrt{h_{t}}, h_{t}=\alpha_{0}+\sum_{i=1}^{r}\alpha_{i}\varepsilon_{t-i}^{2}+\sum_{j=1}^{s}\beta_{j}h_{t-j}, \end{eqnarray} $ (2.2)

其中 $B$为滞后算子, $d$是分数差分参数, $-1/2<d<1/2$, $y_{t}$ $t$时刻的对数收益率, $\Phi(B)$ $\Psi(B)$分别是 $p$阶和 $q$阶的滞后多项式, $\Phi(B)=1-\phi_{1}B-\phi_{2}B-\cdots-\phi_{p}B^{p}$, $\Psi(B)=1+\psi_{1}B+\psi_{2}B\cdots+\psi_{q}B^{q}$, $(1-B)^{d}=\sum\limits_{k=0}^{\infty}\frac{k-d-1)!}{k!(-d-1)!}B^{k}$, $\varepsilon_{t}$是信息序列, 且均值为0, $\{\eta_{t}\}$是均值为零的独立同分布随机变量序列, $\alpha_{0}>0$, $\alpha_{i}\geq0(i=1, 2, \cdots, r)$, $\beta_{j}\geq0(j=1, 2, \cdots, s)$, 对所有的 $t$都有 $\eta_{t}$ $\varepsilon_{t}$是相互独立的, 即称随机过程 $\{\varepsilon_{t}\}$为ARFIMA $(p, d, q)$-GARCH $(r, s)$模型.

3 混成检验

$\theta=(\gamma^{'}, \delta^{'})^{'}$是ARFIMA $(p, q, d)$-GARCH $(r, s)$模型的未知参数, 它的真实值是 $\theta_{0}$, 其中 $\gamma=\left(d, \phi_{1}, \phi_{2}, \cdots, \phi_{p}, \psi_{1}, \psi_{2}, \cdots, \psi_{q}\right)^{'}$, $\delta=\left(\alpha_{0}, \cdots, \alpha_{r}, \beta_{1}, \cdots, \beta_{s}\right)^{'}$.令 $l=p+q+r+s+2$, 那么 $\theta$是一个 $l$维向量, 参数空间为 $\Theta=\Theta_{\gamma}\times\Theta_{\delta}$, 其中 $\Theta_{\gamma}\subset\mathbb{R}^{p+q+1}$, $\Theta_{\delta}\subset\mathbb{R}^{r+s+1}_{0}$, $\mathbb{R}=\left(-\infty, +\infty\right)$, $\mathbb{R}_{0}=\left[0, +\infty\right)$.设 $\theta_{0}$ $\Theta$中的一个内点, 定义 $\alpha(B)=\sum\limits_{i=1}^{r}\alpha_{i}B^{i}$, $\beta(B)=1-\sum\limits_{j=1}^{s}\beta_{j}B^{j}$.

假设1   $-1/2<d<1/2$, 对于每个 $\theta\in\Theta$, 多项式 $\phi(B)$ $\psi(B)$的所有根都在单位圆之外, 当 $\phi_{p}\neq0$或者 $\psi_{q}\neq0$时, $\phi(B)$ $\psi(B)$没有公共根.

假设2  对于每个 $\theta\in\Theta$, $\alpha(B)$ $\beta(B)$没有公共根, $\alpha(1)\neq0$, $\alpha_{r}+\beta_{s}\neq0$, $\sum\limits_{j=1}^{s}\beta_{j}<1$.

假设1表明了 $\{y_{t}\}$的平稳可逆性, 当 $-1/2<d<0$时, ${y_{t}}$表现出间断记忆性, 然而当 $0<d<1/2$时, ${y_{t}}$表现出长期记忆性.假设2是模型(2.2) 的可识别性条件.给出观测值 $\{y_{n}, \cdots, y_{1}\}$和初始值 $Y_{0}\equiv\{y_{0}, y_{-1}, \cdots\}$, 写参数模型如下

$ \begin{align} & \Phi (B){{(1-B)}^{d}}{{y}_{t}}=\Psi (B){{\varepsilon }_{t}}(\gamma ), \\ & {{\varepsilon }_{t}}(\gamma )={{\eta }_{t}}(\theta )\sqrt{{{h}_{t}}(\theta )},{{h}_{t}}(\theta )={{\alpha }_{0}}+\sum\limits_{i=1}^{r}{{{\alpha }_{i}}}\varepsilon _{t-i}^{2}(\theta )+\sum\limits_{j=1}^{s}{{{\beta }_{j}}}{{h}_{t-j}}(\theta ). \\ \end{align} $

$\hat{\theta}_{n}=(\hat{\gamma}_{n}^{'}, \hat{\delta}_{n}^{'})^{'}$, 由文献[7]可知 $\theta_{0}$的拟极大指数似然估计被定义为

$ {{{\hat{\theta }}}_{n}}=\arg \underset{\theta \in \Theta }{\mathop{\rm{min}}}\,{{L}_{n}}(\theta ),{{L}_{n}}(\theta )=\sum\limits_{t=1}^{n}{\left[ \log \sqrt{{{h}_{t}}(\theta )}+\frac{|{{\varepsilon }_{t}}(\gamma )|}{\sqrt{{{h}_{t}}(\theta )}} \right]}. $

假设3   $\eta_{t}$的中位数等于0, $E|\eta_{t}|=1$, ${\rm Var}(\eta_{t}^2)=\sum\limits_{t=1}^{n}\eta_{t}^2<\infty$, $\eta_{t}$的概率密度函数满足 $f(0)>0, \sup\limits_{x\in \mathbb{R}}f(x)<\infty$, 且在0处是连续的.

假设4   $\varepsilon_{t}$是严平稳遍历过程, 且 $E\varepsilon_{t}^2<\infty$.

假设5   $\sqrt{n}(\hat{\theta}_{n}-\theta_{0})=O_{p}(1)$.

假设3是LAD类估计的一般性条件, 见文献[4].假设4的充分必要条件是 $\sum\limits_{i=1}^{r}\alpha_{i}+\sum\limits_{j=1}^{s}\beta_{j}<1$, 见文献[3].假设5是为了简化的证明.在假设1-4下, 由文献[7]可知

$ \sqrt{n}\left(\hat{\theta}_n-\theta_0\right)\rightarrow_d\left(0, \frac{1}{4}\Sigma^{-1}_0\Omega_{0}\Sigma^{-1}_0\right), $

其中

$ \begin{array}{l} {\Omega _0} = E\left[ {\frac{1}{{{h_t}({\theta _0})}}\frac{{\partial {\varepsilon _t}({\gamma _0})}}{{\partial \theta }}\frac{{\partial {\varepsilon _t}({\gamma _0})}}{{\partial {\theta ^T}}}} \right] + \frac{{E\eta _t^2 - 1}}{4}E\left[ {\frac{1}{{h_t^2({\theta _0})}}\frac{{\partial {h_t}({\theta _0})}}{{\partial \theta }}{{\frac{{\partial {h_t}({\theta _0})}}{{\partial \theta }}}^\mathit{T}}} \right],\\ {\Sigma _0} = f(0)E\left[ {\frac{1}{{{h_t}({\theta _0})}}\frac{{\partial {\varepsilon _t}({\gamma _0})}}{{\partial \theta }}\frac{{\partial {\varepsilon _t}({\gamma _0})}}{{\partial {\theta ^T}}}} \right] + \frac{1}{8}E\left[ {\frac{1}{{h_t^2({\theta _0})}}\frac{{\partial {h_t}({\theta _0})}}{{\partial \theta }}{{\frac{{\partial {h_t}({\theta _0})}}{{\partial \theta }}}^T}} \right]. \end{array} $

定义残差 $\hat{\eta}_{t}=\eta_{t}(\hat{\theta}_{n})$, 那么滞后 $\iota$平方残差自相关函数被定义为

$ \hat{\rho}_{\iota}^{*} =\frac{\sum\limits_{t=\iota+1}^{n}(\hat{\eta}_{t}^2-\bar{\eta})(\hat{\eta}_{t-\iota}^2-\bar{\eta})}{\sum\limits_{t=1}^{n}(\hat{\eta}_{t}^2-\bar{\eta})^{2}}, $

其中 $\bar{\eta}=\frac{1}{n}\sum\limits_{t=1}^{n}\hat{\eta}_{t}^2$, 由假设5可知, $\hat{\theta}_{n}-\theta_{0}=o_{p}(1)$.在假设1-4下, 在文献[8]中由定理3.1和控制收敛定理, 能够表明

$ \bar{\eta}=\mu+o_{p}(1), \frac{1}{n}\sum\limits_{t=\iota+1}^{n}(\hat{\eta}_{t}^2-\bar{\eta})^{2}=\sigma_{0}^{2}+o_{p}(1), $

其中 $\mu=E\eta_{t}^2$, 因此理论上只需要考虑

$ \hat{\rho}_{\iota}=\frac{1}{n\sigma_{0}^{2}}\sum\limits_{t=\iota+1}^{n}(\hat{\eta}_{t}^2-\mu)(\hat{\eta}_{t-\iota}^2-\mu). $ (3.1)

$C=\left(C_{1}, C_{2}, \cdots, C_{M}\right)^{'}$, $\hat{C}=\left(\hat{C_{1}}, \hat{C_{2}}, \cdots, \hat{C_{M}}\right)^{'}$, 其中 $M$为正整数, $\iota=1, 2, \cdots, M$, $C_{\iota}=\frac{1}{n}\sum\limits_{t=\iota+1}^{n}\left(\eta_{t}^2-\mu\right)\left(\eta_{t-\iota}^2-\mu\right)$, $\hat{C}_{\iota}=\frac{1}{n}\sum\limits_{t=\iota+1}^{n}\left(\hat{\eta}_{t}^2-\mu)(\hat{\eta}_{t-\iota}^2-\mu\right)$, 由泰勒展示, 有

$ \hat{C}\simeq C+\frac{\partial C}{\partial \theta}\left(\hat{\theta}_{n}-\theta_{0}\right), $ (3.2)

其中 $\partial C/\partial \theta=\left(\partial C_{1}/\partial \theta, \partial C_{2}/\partial \theta, \cdots, \partial C_{m}/\partial \theta\right)^{'}$,

$ \begin{align} & \frac{\partial {{C}_{\iota }}}{\partial \theta }=\frac{1}{n}\sum\limits_{t=\iota +1}^{n}{\frac{\partial \left( \eta _{t}^{2}-\mu \right)}{\partial \theta }}(\eta _{t-\iota }^{2}-\mu )+\frac{1}{n}\sum\limits_{t=\iota +1}^{n}{(\eta _{t}^{2}-\mu )}\frac{\partial (\eta _{t-\iota }^{2}-\mu )}{\partial \theta } \\ & \ \ \ \ \ \ \ \ =\frac{1}{n}\sum\limits_{t=\iota +1}^{n}{\frac{\partial (\frac{\varepsilon _{t}^{2}}{{{h}_{t}}}-\mu )}{\partial \theta }}\left( \frac{\varepsilon _{t-\iota }^{2}}{{{h}_{t}}}-\mu \right)+\frac{1}{n}\sum\limits_{t=\iota +1}^{n}{\left( \frac{\varepsilon _{t}^{2}}{{{h}_{t}}}-\mu \right)}\frac{\partial (\frac{\varepsilon _{t-\iota }^{2}}{{{h}_{t-\iota }}}-\mu )}{\partial \theta } \\ & \ \ \ \ \ \ \ \ =\frac{1}{n}\sum\limits_{t=\iota +1}^{n}{\left( \frac{2{{\varepsilon }_{t}}}{{{h}_{t}}}\frac{\partial {{\varepsilon }_{t}}}{\partial \theta }-\frac{\varepsilon _{t}^{2}}{h_{t}^{2}}\frac{\partial {{h}_{t}}}{\partial \theta } \right)}\left( \frac{\varepsilon _{t-\iota }^{2}}{{{h}_{t-\iota }}}-\mu \right) \\ & \ \ \ \ \ \ \ \ \ \ \ +\frac{1}{n}\sum\limits_{t=\iota +1}^{n}{\left( \frac{\varepsilon _{t}^{2}}{{{h}_{t}}}-\mu \right)}\left( \frac{2{{\varepsilon }_{t-\iota }}}{{{h}_{t-\iota }}}\frac{\partial {{\varepsilon }_{t-\iota }}}{\partial \theta }-\frac{\varepsilon _{t-\iota }^{2}}{h_{t-\iota }^{2}}\frac{\partial {{h}_{t-\iota }}}{\partial \theta } \right) \\ & \ \ \ \ \ \ \ \ =\frac{1}{n}\sum\limits_{t=\iota +1}^{n}{\frac{2{{\varepsilon }_{t}}}{{{h}_{t}}}}\frac{\partial {{\varepsilon }_{t}}}{\partial \theta }\left( \frac{\varepsilon _{t-\iota }^{2}}{{{h}_{t-\iota }}}-\mu \right)-\frac{1}{n}\sum\limits_{t=\iota +1}^{n}{\frac{\varepsilon _{t}^{2}}{h_{t}^{2}}}\frac{\partial {{h}_{t}}}{\partial \theta }\left( \frac{\varepsilon _{t-\iota }^{2}}{{{h}_{t-\iota }}}-\mu \right) \\ &\ \ \ \ \ \ \ \ \ \ \ +\frac{1}{n}\sum\limits_{t=\iota +1}^{n}{\left( \frac{\varepsilon _{t}^{2}}{{{h}_{t}}}-\mu \right)}\frac{2{{\varepsilon }_{t-\iota }}}{{{h}_{t-\iota }}}\frac{\partial {{\varepsilon }_{t-\iota }}}{\partial \theta }-\frac{1}{n}\sum\limits_{t=\iota +1}^{n}{\left( \frac{\varepsilon _{t}^{2}}{{{h}_{t}}}-\mu \right)}\frac{\varepsilon _{t-\iota }^{2}}{h_{t-\iota }^{2}}\frac{\partial {{h}_{t-\iota }}}{\partial \theta }, \\ \end{align} $

由遍历定理可知, 上式中最后一个等式的第一、三、四项都为 $0$, 因此有

$ \frac{\partial C_{\iota}}{\partial \theta}=-\frac{1}{n}\sum\limits_{t=\iota+1}^n\frac{\varepsilon_{t}^2}{h_{t}^2}\frac{\partial h_{t}}{\partial \theta}\left(\frac{\varepsilon_{t-\iota}^2}{h_{t-\iota}}-\mu\right)=-\frac{1}{n}\sum\limits_{t=\iota+1}^n\frac{\eta_{t}^2\left(\eta_{t-\iota}^2-\mu\right)}{h_{t}}\frac{\partial h_{t}}{\partial \theta}, $

因此当 $n\rightarrow \infty$时, $ \frac{\partial C_{\iota}}{\partial \theta}\overset{\text{a.s.}}{\longrightarrow}-\mu X_{\rho\iota}, $其中 $X_{\rho\iota}=E\left[\frac{\eta_{t-\iota}^2-\mu}{h_{t}}\frac{\partial h_{t}}{\partial \theta}\right]$.令 $X_{\rho}=\left(X_{\rho1}, X_{\rho2}, \cdots, X_{\rho M}\right)^{'}$, 则等式(3.2) 可写成

$ \hat{C}\simeq C+\left(-\mu X_{\rho}\right)\left(\hat{\theta}_{n}-\theta_{0}\right). $ (3.3)

$\hat{\rho}=\left(\hat{\rho}_{1}, \hat{\rho}_{2}, \cdots, \hat{\rho}_{M}\right)^T$, $\rho=\left(\rho_{1}, \rho_{2}, \cdots, \rho_{M}\right)^T$, 由等式(3.1) 和等式(3.3), 有

$ \sqrt{n}\hat{\rho}=\sqrt{n}\rho+\sigma_{0}^{-2}\left(-\mu X_{\rho}\right)\sqrt{n}\left(\hat{\theta}_{n}-\theta_{0}\right)+o_{p}(1). $ (3.4)

定理1  如果假设1-5成立, 那么

$ \sqrt{n}\hat{\rho}=\left(\hat{\rho}_{1}, \hat{\rho}_{2}, \cdots, \hat{\rho}_{M}\right)^{'}\longrightarrow_{d}N\left(0, V\right), $

其中

$ \begin{align} & V={{1}_{M}}+\frac{{{\mu }^{2}}}{4\sigma _{0}^{4}}{{X}_{\rho }}\Sigma _{0}^{-1}{{\Omega }_{0}}\Sigma _{0}^{-1}X_{\rho }^{T}-\frac{\mu {{\kappa }_{1}}}{2\sigma _{0}^{4}}{{X}_{\rho }}\Sigma _{0}^{-1}X_{\rho }^{T}+\frac{\mu {{\kappa }_{2}}}{2\sigma _{0}^{4}}\left( X_{\rho }^{*}\Sigma _{0}^{-1}X_{\rho }^{T}+X_{\rho }^{T}\Sigma _{0}^{-1}X_{\rho }^{*} \right), \\ & X_{\rho }^{*}={{\left( X_{\rho 1}^{*}, X_{\rho 2}^{*}, \cdots, X_{\rho M}^{*} \right)}^{\mathit{'}}}, X_{\rho \iota }^{*}=E\left[\frac{\eta _{t-\iota }^{2}-\mu }{\sqrt{{{h}_{t}}}}\frac{\partial {{\varepsilon }_{t}}({{\theta }_{0}})}{\partial \theta } \right], \\ & {{\kappa }_{1}}=E\left[\eta _{t}^{2}(|{{\eta }_{t}}|-1) \right], {{\kappa }_{2}}=E\left[\rm{sgn}({{\eta }_{t}})\eta _{t}^{2} \right]. \\ \end{align} $

  由文献[9]中的定理2.8.1, 能够得到 $ \sqrt{n}C\longrightarrow_{d}N\left(0, \sigma_{0}^{4}1_{M}\right), $其中 $l_{M}$ $M\times M$阶单位矩阵.令

$ D_{t}=\frac{|\eta_{t}|-1}{4h_{t}}\frac{\partial h_{t}(\theta_0)}{\partial \theta}-\frac{{\rm sgn}(\eta_{t})}{2\sqrt{h_t}}\frac{\partial \varepsilon_t(\theta_0)}{\partial \theta}, $

由文献[3]可知

$ \sqrt{n}\left(\hat{\theta}_n-\theta_0\right)=\frac{\Sigma_0^{-1}}{\sqrt{n}}\sum\limits_{t=1}^nD_{t}+o_p(1), $

因此由Mann-Wald device和鞅差中心极限定理[10]可知

$ \begin{align} & \ \ \ \rm{Cov}\left( \sqrt{\mathit{n}}\rm{(}{{{\mathit{\hat{\theta }}}}_{\mathit{n}}}\mathit{-}{{\mathit{\theta }}_{\rm{0}}}\rm{)}\mathit{,}\sqrt{\mathit{n}}\mathit{C} \right)=\mathit{E}\left[ \sqrt{\mathit{n}}\rm{(}{{{\mathit{\hat{\theta }}}}_{\mathit{n}}}\mathit{-}{{\mathit{\theta }}_{\rm{0}}})\cdot \sqrt{\mathit{n}}{{\mathit{C}}^{\mathit{T}}} \right] \\ & =\mathit{E}\left[ \frac{\Sigma _{0}^{-1}}{\sqrt{\mathit{n}}}\sum\limits_{\mathit{t}=1}^{\mathit{n}}{{{\mathit{D}}_{\mathit{t}}}}\cdot \sqrt{\mathit{n}}{{\mathit{C}}^{\mathit{T}}} \right]=\Sigma _{0}^{-1}\mathit{E}\left[ \sum\limits_{\mathit{t}=1}^{\mathit{n}}{{{\mathit{D}}_{t}}}\cdot {{\mathit{C}}^{\mathit{T}}} \right], \\ \end{align} $

$ \begin{array}{l} E[\sum\limits_{t = 1}^n {{D_t}} \cdot {C_\iota }] = E\left[ {\sum\limits_{t = 1}^n {\left( {\frac{{|{\eta _t}| - 1}}{{4{h_t}}}\frac{{\partial {h_t}({\theta _0})}}{{\partial \theta }} - \frac{{{\rm{sgn}}({\eta _t})}}{{2\sqrt {{h_t}} }}\frac{{\partial {\varepsilon _t}({\theta _0})}}{{\partial \theta }}} \right)} \cdot } \right.\\ \;\;\;\;\;\;\;\;\;\;\;\;\;\;\;\;\;\;\;\;\;\left. {\;\frac{1}{n}\sum\limits_{s = \iota + 1}^n {\left( {\eta _s^2 - \mu } \right)} \left( {\eta _{s - \iota }^2 - \mu } \right)} \right]\\ \;\;\;\;\;\;\;\;\;\;\;\;\;\;\;\;\;\; = \frac{1}{n}E\left[ {\sum\limits_{t = 1}^n {\left( {\frac{{|{\eta _t}| - 1}}{{4{h_t}}}\frac{{\partial {h_t}({\theta _0})}}{{\partial \theta }}} \right)} \cdot \sum\limits_{s = \iota + 1}^n {\left( {\eta _s^2 - \mu } \right)} \left( {\eta _{s - \iota }^2 - \mu } \right)} \right]\\ \;\;\;\;\;\;\;\;\;\;\;\;\;\;\;\;\;\;\;\;\;\; - \frac{1}{n}E\left[ {\frac{{{\rm{sgn}}({\eta _t})}}{{2\sqrt {{h_t}} }}\frac{{\partial {\varepsilon _t}({\theta _0})}}{{\partial \theta }}) \cdot \sum\limits_{s = \iota + 1}^n {\left( {\eta _s^2 - \mu } \right)} \left( {\eta _{s - \iota }^2 - \mu } \right)} \right]\\ \;\;\;\;\;\;\;\;\;\;\;\;\;\;\;\;\;\; = \frac{1}{n}\sum\limits_{t = 1}^n {\sum\limits_{s = \iota + 1}^n E } \left[ {\frac{{|{\eta _t}| - 1}}{{4{h_t}}}\frac{{\partial {h_t}({\theta _0})}}{{\partial \theta }}\left( {\eta _s^2 - \mu } \right)\left( {\eta _{s - \iota }^2 - \mu } \right)} \right]\\ \;\;\;\;\;\;\;\;\;\;\;\;\;\;\;\;\;\;\;\;\; - \frac{1}{n}\sum\limits_{t = 1}^n {\sum\limits_{s = \iota + 1}^n E } \left[ {\frac{{{\rm{sgn}}({\eta _t})}}{{2\sqrt {{h_t}} }}\frac{{\partial {\varepsilon _t}({\theta _0})}}{{\partial \theta }}\left( {\eta _s^2 - \mu } \right)\left( {\eta _{s - \iota }^2 - \mu } \right)} \right]\\ \;\;\;\;\;\;\;\;\;\;\;\;\;\;\;\;\; = \frac{1}{n}\sum\limits_{t = 1}^n E \left[ {\frac{{|{\eta _t}| - 1}}{{4{h_t}}}\frac{{\partial {h_t}({\theta _0})}}{{\partial \theta }}\left( {\eta _t^2 - \mu } \right)\left( {\eta _{t - \iota }^2 - \mu } \right)} \right]\\ \;\;\;\;\;\;\;\;\;\;\;\;\;\;\;\;\;\;\;\; - \frac{1}{n}\sum\limits_{t = 1}^n E \left[ {\frac{{{\rm{sgn}}({\eta _t})}}{{2\sqrt {{h_t}} }}\frac{{\partial {\varepsilon _t}({\theta _0})}}{{\partial \theta }}\left( {\eta _t^2 - \mu } \right)\left( {\eta _{t - \iota }^2 - \mu } \right)} \right]\\ \;\;\;\;\;\;\;\;\;\;\;\;\;\;\;\;\; = E\left[ {\frac{{|{\eta _t}| - 1}}{{4{h_t}}}\frac{{\partial {h_t}({\theta _0})}}{{\partial \theta }}\left( {\eta _t^2 - \mu } \right)\left( {\eta _{t - \iota }^2 - \mu } \right)} \right]\\ \;\;\;\;\;\;\;\;\;\;\;\;\;\;\;\;\;\;\;\; - E\left[ {\frac{{{\rm{sgn}}({\eta _t})}}{{2\sqrt {{h_t}} }}\frac{{\partial {\varepsilon _t}({\theta _0})}}{{\partial \theta }}\left( {\eta _t^2 - \mu } \right)\left( {\eta _{t - \iota }^2 - \mu } \right)} \right], \end{array} $

$ E[\sum\limits_{t=1}^{n}D_t\cdot C_\iota] =\frac{\kappa_1}{4}X_{\rho\iota}-\frac{\kappa_2}{2}X_{\rho\iota}^*, $那么 $ {\rm Cov}\left({\sqrt{n}(\hat{\theta}_n-\theta_0), \sqrt{n}C^T}\right)=\Sigma_0^{-1}E\left[\frac{\kappa_1}{4}X_{\rho}-\frac{\kappa_2}{2}X_{\rho}^*\right], $因此 $\sqrt{n}\rho$的协方差为

$ \begin{eqnarray*} &&{\rm var}\left(\sqrt{n}\rho\right)=\sigma_0^{-4}{\rm var}(\sqrt{n}\hat{C})\\ &=&\sigma_0^{-4}[{\rm var}\left(\sqrt{n}C\right)+{\rm var}\left(-\mu X_\rho\sqrt{n} \left(\hat{\theta}_n-\theta_0\right)\right)\\ &&+{\rm cov}\left(\sqrt{n}C, -\mu X_\rho\sqrt{n}\left(\hat{\theta}_n-\theta_0\right)\right) +{\rm cov}\left(-\mu X_\rho\sqrt{n}\left(\hat{\theta}_n-\theta_0\right), \sqrt{n}C\right)]\\ &=&\sigma_0^{-4}[\sigma_0^{4}1_M+\mu^2X_\rho\frac{1}{4}\Sigma_0^{-1}\Omega_0\Sigma_0^{-1}X_\rho^{T} -\mu\left(\frac{\kappa_1}{4}X_\rho-\frac{\kappa_2}{2}X_\rho^{*}\right)\Sigma_0^{-1}X_\rho^{T}\\ &&-\mu X_\rho^{T}\Sigma_0^{-1}\left(\frac{\kappa_1}{4}X_\rho-\frac{\kappa_2}{2}X_\rho^{*}\right)]\\ &=&1_{M}+\frac{\mu^2}{4\sigma_{0}^{4}}X_{\rho}\Sigma_{0}^{-1}\Omega_{0}\Sigma_{0}^{-1}X_{\rho}^T -\frac{\mu \kappa_{1}}{2\sigma_{0}^{4}}X_{\rho}\Sigma_{0}^{-1}X_{\rho}^T +\frac{\mu \kappa_{2}}{2\sigma_{0}^{4}} \left(X_{\rho}^{*}\Sigma_{0}^{-1}X_{\rho}^T+X_{\rho}^T\Sigma_{0}^{-1}X_{\rho}^{*}\right), \end{eqnarray*} $

即完成了定理的证明.

在定理1中, 通过样本均值来估计 $V$, 记为 $\hat{V}$, 在假设1-4下, 表明 $ \hat{V}=V+o_p(1), $因此由定理1, 下面的结论是直接成立的.

结论1  如果假设1-5成立, 那么当 $n\rightarrow\infty$时, $ Q\left(M\right)=n\hat{\rho}^T\hat{V}^{-1}\hat{\rho}\longrightarrow_d\chi^2(M). $对于拟极大指数似然估计, 在结论1中称 $Q\left(M\right)$为基于平方残差自相关函数的混成检验统计量, 并且可用 $Q\left(M\right)$来诊断检验由拟极大指数似然估计拟合的ARFIMA-GARCH模型.

4 实例分析

在拟极大指数似然估计下, 为验证基于平方残差自相关函数的混成检验统计量的有效性, 以中证800指数的股票价格为例, 选取从2007年1月4日到2008年12月31日共488个数据(数据来自http://quotes.money.163.com/1399906.html).根据ADF检验法, 可知该序列是平稳的.由图 1图 2可以看出, 序列显然存在异方差性.通过MATLAB软件, 利用R/S检验法可以计算出中证800指数收益率序列的Hurst指数, 即 $H=0.5405$, 由于 $H>0.5$, 因此该序列存在长记忆性.由以上可知, 对该序列建立ARFIMA(1, $d$, 0)-GARCH(1, 1) 模型, 使用拟极大指数似然估计方法, 得到的拟合模型为

图 1 中证800指数原始数据图

图 2 中证800指数对数收益率图
$ \begin{align} & (1-0.0427B){{(1-B)}^{0.0003}}{{y}_{t}}={{\varepsilon }_{t}}, \\ & {{\varepsilon }_{t}}={{\eta }_{t}}\sqrt{{{h}_{t}}}, {{h}_{t}}=0.0836+0.2349\varepsilon _{t-1}^{2}+0.5000{{h}_{t-1}}. \\ \end{align} $

然而, 在实际中拟合的模型是否有误差, 是否真正符合实际数据, 就需要对拟合后的模型进行诊断检验.因此, 可利用基于平方残差自相关函数的混成检验统计量( $Q\left(M\right)$)对上述拟合模型进行诊断检验.由卡方分布表可知, 在0.05显著性水平下, $\chi^2(6)=12.592$, $\chi^2(12)=21.026$.通过MTALAB软件计算可得, $Q\left(6\right)=4.3431<\chi^2(6)$, $Q\left(12\right)=8.2062<\chi^2(12)$.从而可以看出, 上述拟合的ARFIMA(1, $d$, 0)-GARCH(1, 1) 模型是准确的, 符合实际数据, ARFIMA(1, $d$, 0)-GARCH(1, 1) 模型适合于拟合该时间段内的中证800指数收益率序列.

参考文献
[1] 彭静, 刘剑锋, 王晓天. 股票收益的分形特征的实证分析[J]. 数学杂志, 2005, 25(5): 579–582.
[2] Hosking J R M. Fractional differencing[J]. Biometrika, 1981, 68(1): 165–176. DOI:10.1093/biomet/68.1.165
[3] Bollerslev T. Generalized autoregressive conditional heteroskedasticity[J]. J. Econ., 1986, 31(3): 307–327. DOI:10.1016/0304-4076(86)90063-1
[4] Li G D, Li W K. Least absolute deviation estimation for fractionally integrated autoregressive moving average time series models with conditional heteroscedasticity[J]. Biometrika, 2008, 95(2): 399–414. DOI:10.1093/biomet/asn014
[5] Zhu K. A mixed portmanteau test for ARMA-GARCH models by the quasi-maximum exponential likelihood estimation approach[J]. J. Time Ser. Anal., 2013, 34(2): 230–237. DOI:10.1111/jtsa.2013.34.issue-2
[6] Wong H, Ling S Q. Mixed portmanteau tests for time series models[J]. J. Time Ser. Anal., 2005, 26(4): 569–579. DOI:10.1111/jtsa.2005.26.issue-4
[7] Pan B G, Chen M. Self-weighted quasi-maximum exponential likelihood estimator for ARFIMA-GARCH models[J]. J. Stat. Plan. Infer., 2013, 143(4): 716–729. DOI:10.1016/j.jspi.2012.10.002
[8] Ling S Q, Mcaleer M. Asymptotic theory for a new vector ARMA-GARCH model[J]. Econ. The., 2003, 19(2): 280–310.
[9] Lehmann E L. Elements of large-sample theory[M]. New York: Springer, 1998.
[10] Billingsley P. The linderberg-levy theorem for martingales[J]. Proc. Amer. Math. Soc., 1961, 12(5): 788–792.